Những câu hỏi liên quan
๖²⁴ʱ乂ų✌й๏✌ρɾ๏༉
Xem chi tiết
Ngoc An Pham
Xem chi tiết
Hồng Phúc
6 tháng 2 2021 lúc 9:36

\(\dfrac{y-z}{\left(x-y\right)\left(x-z\right)}+\dfrac{z-x}{\left(y-z\right)\left(y-x\right)}+\dfrac{x-y}{\left(z-x\right)\left(z-y\right)}\)

\(=\dfrac{z-x+x-y}{\left(x-y\right)\left(z-x\right)}+\dfrac{x-y+y-z}{\left(y-z\right)\left(x-y\right)}+\dfrac{y-z+z-x}{\left(z-x\right)\left(y-z\right)}\)

\(=\dfrac{1}{x-y}+\dfrac{1}{z-x}+\dfrac{1}{y-z}+\dfrac{1}{x-y}+\dfrac{1}{z-x}+\dfrac{1}{y-z}\)

\(=\dfrac{2}{x-y}+\dfrac{2}{y-z}+\dfrac{2}{z-x}\)

Trần Thị Hảo
Xem chi tiết
Nguyễn Lê Phước Thịnh
26 tháng 11 2022 lúc 13:59

a: \(\dfrac{y}{\left(x-y\right)\left(y-z\right)}-\dfrac{z}{\left(y-z\right)\left(x-z\right)}-\dfrac{x}{\left(x-y\right)\left(x-z\right)}\)

\(=\dfrac{xy-yz-xz+yz-xy+xz}{\left(x-y\right)\left(y-z\right)\left(x-z\right)}\)

=0

c: \(=\dfrac{1}{x\left(x-y\right)\left(x-z\right)}-\dfrac{1}{y\left(y-z\right)\left(x-y\right)}+\dfrac{1}{z\left(x-z\right)\left(y-z\right)}\)

\(=\dfrac{zy\left(y-z\right)-xz\left(x-z\right)+xy\left(x-y\right)}{xyz\left(x-y\right)\left(y-z\right)\left(x-z\right)}\)

\(=\dfrac{zy^2-z^2y-x^2z+xz^2+xy\left(x-y\right)}{xyz\left(x-y\right)\left(y-z\right)\left(x-z\right)}\)

\(=\dfrac{1}{xyz}\)

 

l҉o҉n҉g҉ d҉z҉
Xem chi tiết
KCLH Kedokatoji
9 tháng 9 2020 lúc 22:00

Động não tí đi Quỳnh, a thấy bài này cũng không khó.

Khách vãng lai đã xóa
Khanh Nguyễn Ngọc
9 tháng 9 2020 lúc 22:10

Bài dễ mừ, có phải Croatia thật ko vậy :))  (viết đề bị nhầm, là x,y,z dương chứ :))

Áp dụng Cauchy-Schwarz dạng cộng mẫu số:

\(\frac{x^2}{\left(x+y\right)\left(x+z\right)}+\frac{y^2}{\left(y+z\right)\left(y+x\right)}+\frac{z^2}{\left(z+x\right)\left(z+y\right)}\ge\)

\(\frac{\left(x+y+z\right)^2}{\left(x+y\right)\left(x+z\right)+\left(y+z\right)\left(y+x\right)+\left(z+x\right)\left(z+y\right)}=\frac{\left(x+y+z\right)^2}{x^2+y^2+z^2+3\left(xy+yz+zx\right)}\)

\(=\frac{\left(x+y+z\right)^2}{\left(x+y+z\right)^2+\left(xy+yz+zx\right)}\)

Xét \(xy+yz+zx\le\frac{\left(x+y+z\right)^2}{3}\Rightarrow\frac{\left(x+y+z\right)^2}{\left(x+y+z\right)^2+\left(xy+yz+zx\right)}\ge\frac{\left(x+y+z\right)^2}{\left(x+y+z\right)^2+\frac{\left(x+y+z\right)^2}{3}}\)

\(=\frac{\left(x+y+z\right)^2}{\frac{4}{3}\left(x+y+z\right)^2}=\frac{3}{4}\)

Dấu bằng xảy ra khi và chỉ khi x=y=z,  Xong! :))

Khách vãng lai đã xóa
Agami Raito
Xem chi tiết
trần xuân quyến
Xem chi tiết
Nguyễn__ Hiền 6
Xem chi tiết
Nguyễn__ Hiền 6
3 tháng 12 2017 lúc 20:35

bằng 0 thôi nhé

yume nịjume
3 tháng 12 2017 lúc 20:37

bằng 0

Sách Giáo Khoa
Xem chi tiết
Nguyen Thuy Hoa
28 tháng 6 2017 lúc 16:07

Phép cộng các phân thức đại số

Phép cộng các phân thức đại số

Hoàng
Xem chi tiết